Stronger version of AMM problem 11145 (April 2005)?












39












$begingroup$


How to show that for $a_1,a_2,cdots,a_n >0$ real numbers and for $n ge 3$:



$$sum_{k=1}^{n}dfrac{k}{a_{1}+a_{2}+cdots+a_{k}}leleft(2-dfrac{7ln{2}}{8ln{n}}right)sum_{k=1}^{n}dfrac{1}{a_{k}}$$



This version seems stronger than the inequality mentioned here.



Addition: A sister problem: For $a_1,a_2,cdots,a_n >0$ real numbers and for $n ge 2$, we have the version:



$$displaystyle dfrac{1}{1+a_{1}}+dfrac{1}{1+a_{1}+a_{2}}+cdots+dfrac{1}{1+a_{1}+a_{2}+cdots+a_{n}} le sqrt{dfrac{1}{a_{1}}+dfrac{1}{a_{2}}+cdots+dfrac{1}{a_{n}}}$$



The stronger vesrion claims that: For each $n$, $c_n = left(1-dfrac{ln{n}}{2n}right)$ we have:
$$displaystyle dfrac{1}{1+a_{1}}+dfrac{1}{1+a_{1}+a_{2}}+cdots+dfrac{1}{1+a_{1}+a_{2}+cdots+a_{n}} le c_nsqrt{dfrac{1}{a_{1}}+dfrac{1}{a_{2}}+cdots+dfrac{1}{a_{n}}}$$










share|cite|improve this question











$endgroup$








  • 4




    $begingroup$
    @Downvoters .. care to explain ? =P
    $endgroup$
    – r9m
    Jul 1 '14 at 16:18






  • 2




    $begingroup$
    How did you come up with the term $dfrac{7log(2)}{8log(n)}$?
    $endgroup$
    – robjohn
    Jul 1 '14 at 19:33






  • 8




    $begingroup$
    This holds for $n=2$.
    $endgroup$
    – robjohn
    Jul 1 '14 at 21:21






  • 2




    $begingroup$
    @r9m please, where on earth did that constant surface from? It could be seriously helpful depending how you found it.
    $endgroup$
    – Shakespeare
    Sep 4 '14 at 14:24






  • 2




    $begingroup$
    math.org.cn/forum.php?mod=viewthread&tid=28961
    $endgroup$
    – math110
    Dec 26 '14 at 12:26
















39












$begingroup$


How to show that for $a_1,a_2,cdots,a_n >0$ real numbers and for $n ge 3$:



$$sum_{k=1}^{n}dfrac{k}{a_{1}+a_{2}+cdots+a_{k}}leleft(2-dfrac{7ln{2}}{8ln{n}}right)sum_{k=1}^{n}dfrac{1}{a_{k}}$$



This version seems stronger than the inequality mentioned here.



Addition: A sister problem: For $a_1,a_2,cdots,a_n >0$ real numbers and for $n ge 2$, we have the version:



$$displaystyle dfrac{1}{1+a_{1}}+dfrac{1}{1+a_{1}+a_{2}}+cdots+dfrac{1}{1+a_{1}+a_{2}+cdots+a_{n}} le sqrt{dfrac{1}{a_{1}}+dfrac{1}{a_{2}}+cdots+dfrac{1}{a_{n}}}$$



The stronger vesrion claims that: For each $n$, $c_n = left(1-dfrac{ln{n}}{2n}right)$ we have:
$$displaystyle dfrac{1}{1+a_{1}}+dfrac{1}{1+a_{1}+a_{2}}+cdots+dfrac{1}{1+a_{1}+a_{2}+cdots+a_{n}} le c_nsqrt{dfrac{1}{a_{1}}+dfrac{1}{a_{2}}+cdots+dfrac{1}{a_{n}}}$$










share|cite|improve this question











$endgroup$








  • 4




    $begingroup$
    @Downvoters .. care to explain ? =P
    $endgroup$
    – r9m
    Jul 1 '14 at 16:18






  • 2




    $begingroup$
    How did you come up with the term $dfrac{7log(2)}{8log(n)}$?
    $endgroup$
    – robjohn
    Jul 1 '14 at 19:33






  • 8




    $begingroup$
    This holds for $n=2$.
    $endgroup$
    – robjohn
    Jul 1 '14 at 21:21






  • 2




    $begingroup$
    @r9m please, where on earth did that constant surface from? It could be seriously helpful depending how you found it.
    $endgroup$
    – Shakespeare
    Sep 4 '14 at 14:24






  • 2




    $begingroup$
    math.org.cn/forum.php?mod=viewthread&tid=28961
    $endgroup$
    – math110
    Dec 26 '14 at 12:26














39












39








39


17



$begingroup$


How to show that for $a_1,a_2,cdots,a_n >0$ real numbers and for $n ge 3$:



$$sum_{k=1}^{n}dfrac{k}{a_{1}+a_{2}+cdots+a_{k}}leleft(2-dfrac{7ln{2}}{8ln{n}}right)sum_{k=1}^{n}dfrac{1}{a_{k}}$$



This version seems stronger than the inequality mentioned here.



Addition: A sister problem: For $a_1,a_2,cdots,a_n >0$ real numbers and for $n ge 2$, we have the version:



$$displaystyle dfrac{1}{1+a_{1}}+dfrac{1}{1+a_{1}+a_{2}}+cdots+dfrac{1}{1+a_{1}+a_{2}+cdots+a_{n}} le sqrt{dfrac{1}{a_{1}}+dfrac{1}{a_{2}}+cdots+dfrac{1}{a_{n}}}$$



The stronger vesrion claims that: For each $n$, $c_n = left(1-dfrac{ln{n}}{2n}right)$ we have:
$$displaystyle dfrac{1}{1+a_{1}}+dfrac{1}{1+a_{1}+a_{2}}+cdots+dfrac{1}{1+a_{1}+a_{2}+cdots+a_{n}} le c_nsqrt{dfrac{1}{a_{1}}+dfrac{1}{a_{2}}+cdots+dfrac{1}{a_{n}}}$$










share|cite|improve this question











$endgroup$




How to show that for $a_1,a_2,cdots,a_n >0$ real numbers and for $n ge 3$:



$$sum_{k=1}^{n}dfrac{k}{a_{1}+a_{2}+cdots+a_{k}}leleft(2-dfrac{7ln{2}}{8ln{n}}right)sum_{k=1}^{n}dfrac{1}{a_{k}}$$



This version seems stronger than the inequality mentioned here.



Addition: A sister problem: For $a_1,a_2,cdots,a_n >0$ real numbers and for $n ge 2$, we have the version:



$$displaystyle dfrac{1}{1+a_{1}}+dfrac{1}{1+a_{1}+a_{2}}+cdots+dfrac{1}{1+a_{1}+a_{2}+cdots+a_{n}} le sqrt{dfrac{1}{a_{1}}+dfrac{1}{a_{2}}+cdots+dfrac{1}{a_{n}}}$$



The stronger vesrion claims that: For each $n$, $c_n = left(1-dfrac{ln{n}}{2n}right)$ we have:
$$displaystyle dfrac{1}{1+a_{1}}+dfrac{1}{1+a_{1}+a_{2}}+cdots+dfrac{1}{1+a_{1}+a_{2}+cdots+a_{n}} le c_nsqrt{dfrac{1}{a_{1}}+dfrac{1}{a_{2}}+cdots+dfrac{1}{a_{n}}}$$







real-analysis inequality






share|cite|improve this question















share|cite|improve this question













share|cite|improve this question




share|cite|improve this question








edited Apr 13 '17 at 12:21









Community

1




1










asked Jul 1 '14 at 15:25









r9mr9m

13.3k24171




13.3k24171








  • 4




    $begingroup$
    @Downvoters .. care to explain ? =P
    $endgroup$
    – r9m
    Jul 1 '14 at 16:18






  • 2




    $begingroup$
    How did you come up with the term $dfrac{7log(2)}{8log(n)}$?
    $endgroup$
    – robjohn
    Jul 1 '14 at 19:33






  • 8




    $begingroup$
    This holds for $n=2$.
    $endgroup$
    – robjohn
    Jul 1 '14 at 21:21






  • 2




    $begingroup$
    @r9m please, where on earth did that constant surface from? It could be seriously helpful depending how you found it.
    $endgroup$
    – Shakespeare
    Sep 4 '14 at 14:24






  • 2




    $begingroup$
    math.org.cn/forum.php?mod=viewthread&tid=28961
    $endgroup$
    – math110
    Dec 26 '14 at 12:26














  • 4




    $begingroup$
    @Downvoters .. care to explain ? =P
    $endgroup$
    – r9m
    Jul 1 '14 at 16:18






  • 2




    $begingroup$
    How did you come up with the term $dfrac{7log(2)}{8log(n)}$?
    $endgroup$
    – robjohn
    Jul 1 '14 at 19:33






  • 8




    $begingroup$
    This holds for $n=2$.
    $endgroup$
    – robjohn
    Jul 1 '14 at 21:21






  • 2




    $begingroup$
    @r9m please, where on earth did that constant surface from? It could be seriously helpful depending how you found it.
    $endgroup$
    – Shakespeare
    Sep 4 '14 at 14:24






  • 2




    $begingroup$
    math.org.cn/forum.php?mod=viewthread&tid=28961
    $endgroup$
    – math110
    Dec 26 '14 at 12:26








4




4




$begingroup$
@Downvoters .. care to explain ? =P
$endgroup$
– r9m
Jul 1 '14 at 16:18




$begingroup$
@Downvoters .. care to explain ? =P
$endgroup$
– r9m
Jul 1 '14 at 16:18




2




2




$begingroup$
How did you come up with the term $dfrac{7log(2)}{8log(n)}$?
$endgroup$
– robjohn
Jul 1 '14 at 19:33




$begingroup$
How did you come up with the term $dfrac{7log(2)}{8log(n)}$?
$endgroup$
– robjohn
Jul 1 '14 at 19:33




8




8




$begingroup$
This holds for $n=2$.
$endgroup$
– robjohn
Jul 1 '14 at 21:21




$begingroup$
This holds for $n=2$.
$endgroup$
– robjohn
Jul 1 '14 at 21:21




2




2




$begingroup$
@r9m please, where on earth did that constant surface from? It could be seriously helpful depending how you found it.
$endgroup$
– Shakespeare
Sep 4 '14 at 14:24




$begingroup$
@r9m please, where on earth did that constant surface from? It could be seriously helpful depending how you found it.
$endgroup$
– Shakespeare
Sep 4 '14 at 14:24




2




2




$begingroup$
math.org.cn/forum.php?mod=viewthread&tid=28961
$endgroup$
– math110
Dec 26 '14 at 12:26




$begingroup$
math.org.cn/forum.php?mod=viewthread&tid=28961
$endgroup$
– math110
Dec 26 '14 at 12:26










1 Answer
1






active

oldest

votes


















10





+100







$begingroup$

Haven't had any progress with the original problem but I have done significant progress on the "sister" problem:



Let $A_k=sumlimits_{i=1}^k a_i $, thus the problem can be re-written as
$$sumlimits_{k=1}^n frac{1}{1+A_k} leq sqrt{sumlimits_{k=1}^n frac{1}{a_k}}$$
or, since both sides are positive,
$$left( sumlimits_{k=1}^n frac{1}{1+A_k} right)^2 leq {sumlimits_{k=1}^n frac{1}{a_k}}$$
Now this is very similiar to the Cauchy-Schwarz inequality, that is, for any positive integers $x_1,x_2...x_n$ and $y_1,y_2...y_n$, the following inequality holds:
$$left( sumlimits_{k=1}^n x_ky_k right)^2 leq left( sumlimits_{k=1}^n x_k^2 right)left( sumlimits_{k=1}^n y_k^2 right)$$
So, assuming $x_k=frac{1}{sqrt{a_k}}$, we gain that $y_k=frac{sqrt{a_k}}{1+A_k}$,
now if $ sumlimits_{k=1}^n y_k^2 =sumlimits_{k=1}^n frac{a_k}{(1+A_k)^2}leq1$
then the original inequality holds. A very nice proof of this due to r9m himself can be found here.



In case of the stronger version
$$sumlimits_{k=1}^n frac{1}{1+A_k} leq c_nsqrt{sumlimits_{k=1}^n frac{1}{a_k}}$$
Using the Cauchy-Schwarz inequality in a similiar fashion we gain that in order for the original inequality to hold,
$$sumlimits_{k=1}^n frac{a_k}{(1+A_k)^2}leq c_n$$ must also hold.
To prove this let us look at the maximal values of $sumlimits_{k=1}^n frac{a_k}{(1+A_k)^2}$. To obtain the maxima we must solve a system of $n$ partial derivatives of this sum equal to zero, or notationally:



begin{cases}
&frac{partial}{partial a_1}sumlimits_{k=1}^n frac{a_k}{(1+A_k)^2}=0 \
&frac{partial}{partial a_2}sumlimits_{k=1}^n frac{a_k}{(1+A_k)^2}=0\
&.\
&.\
&frac{partial}{partial a_n}sumlimits_{k=1}^n frac{a_k}{(1+A_k)^2}=0
end{cases}



Note that if we define $S(i)=sumlimits_{k=i}^n frac{a_k}{(1+A_k)^2}$, then, since no terms of our sum with index less than $i$ contain $a_i$, we may rewrite the system as:
begin{cases}
&frac{partial}{partial a_1}S(1)=0 \
&frac{partial}{partial a_2}S(2)=0\
&.\
&.\
&frac{partial}{partial a_n}S(n)=0
end{cases}



Note that $S(n)= frac{a_n}{(1+A_{n-1}+a_n)^2}$, thus $$frac{partial}{partial a_n}S(n)=frac{1+A_{n-1}-a_n}{(1+A_{n-1}+a_n)^3}$$
Note that it is zero if $a_n=1+A_{n-1}$
Now let us show that in order for maxima to be gained $a_{i}=b_{n-i}(1+A_{i-1})$ for all natural $i<n$ with some sequence $b_i$. So we just gained that $a_{n}=b_0(1+A_{n-2})$ with $b_0=1$.
The maximal value of $S(n)$ is thus
$$max(S(n))=max(frac{a_n}{(1+A_{n-1}+a_n)^2})=frac{b_0(1+A_{n-1})}{(1+A_{n-1}+b_0(1+A_{n-1}))^2}=frac{b_0}{(1+b_0)^2(1+A_{n-1}))}$$



We may now simplify $S(n-1)$, since we know that $a_n=b_0(1+A_{n-1})$,
$$S(n-1)=frac{a_{n-1}}{(1+A_{n-2}+a_{n-1})^2}+S(n)=frac{a_{n-1}}{(1+A_{n-2}+a_{n-1})^2}+frac{a_n}{(1+A_{n-1}+a_n)^2}=frac{a_{n-1}}{(1+A_{n-2}+a_{n-1})^2}+frac{b_0}{(b_0+1)^2(1+A_{n-2}+a_{n-1})}=frac{((b_0+1)^2+b_0)a_{n-1}+b_0(1+A_{n-2})}{(b_0+1)^2(1+A_{n-2}+a_{n-1})^2}$$



So the partial derivative:
$$frac{partial}{partial a_{n-1}}S(n-1)=frac{partial}{partial a_{n-1}}frac{((b_0+1)^2+b_0)a_{n-1}+b_0(1+A_{n-2})}{(b_0+1)^2(1+A_{n-2}+a_{n-1})^2}=frac{((b_0+1)^2-b_0)(1+A_{n-2})-((b_0+1)^2+b_0)a_{n-1}}{(1+b_0)^2(1+A_{n-2}+a_{n-1})^3}$$



Thus for maxima to be atained, $a_{n-1}=b_1(1+A_{n-2})$, and $b_1=frac{(b_0+1)^2-b_0}{(b_0+1)^2+b_0}$ (particularly, $b_1=frac 3 5$),



The maximal value of $S(n-1)$ is then
$$max(S(n-1))=max(frac{((b_0+1)^2+b_0)a_{n-1}+b_0(1+A_{n-2})}{(1+b_0)^2(1+A_{n-2}+a_{n-1})^2})=frac{((b_0+1)^2-b_0)(1+A_{n-2})+b_0(1+A_{n-2})}{(b_0+1)^2(1+A_{n-2}+b_1(1+A_{n-2}))^2}=frac{(b_0+1)^2(1+A_{n-2})}{(b_0+1)^2(b_1+1)^2(1+A_{n-2})^2}=frac{1}{(b_1+1)^2(1+A_{n-2})}$$



$S(n-2)$ can then be calculated as
$$S(n-2)=frac{a_{n-2}}{(1+A_{n-3}+a_{n-2})^2}+S(n-1)=frac{a_{n-2}}{(1+A_{n-3}+a_{n-2})^2}+frac{1}{(b_1+1)^2(1+A_{n-3}+a_{n-2})}=frac{1+A_{n-3}+(1+(b_1+1)^2)a_{n-2}}{(b_1+1)^2(1+A_{n-3}+a_{n-2})^2}$$



The partial derivative is thus:
$$frac{partial}{partial a_{n-2}}S(n-2)=frac{partial}{partial a_{n-2}}frac{1+A_{n-3}+(1+(b_1+1)^2)a_{n-2}}{(b_1+1)^2(1+A_{n-3}+a_{n-2})^2}=frac{b_1(b_1+1)(1+A_{n-3})-(1+(b_1+1)^2)a_{n-2}}{(b_1+1)^2(1+A_{n-3}+a_{n-2})^3}$$



So, once again to attain maxima $a_{n-2}=b_2(1+A_{n-3})$ and $b_2=frac{(b_1+1)^2-1}{(b_1+1)^2+1}$.
And the maximum of $S(n-2)$ is
$$max(S(n-2))=max(frac{1+A_{n-3}+(1+(b_1+1)^2)a_{n-2}}{(b_1+1)^2(1+A_{n-3}+a_{n-2})^2})=frac{((b_1+1)^2-1)(1+A_{n-3})+1+A_{n-3}}{(b_1+1)^2(1+A_{n-3}+b_2(1+A_{n-2}))^2}=frac{(b_1+1)^2(1+A_{n-3})}{(b_1+1)^2(b_2+1)^2(1+A_{n-3})^2}=frac{1}{(b_2+1)^2(1+A_{n-3})}$$



Notice that the expression for the maximum value of $S(n-2)$ is almost identical to the one with $S(n-1)$, only all the indices are shifted by one, thus all the following operations that will be done on higher indices will be analogous, thus we can say that in general
$$max(S(n-i))=frac{1}{(1+b_i)^2(1+A_{n-i-1})}$$
$$b_i=frac{(b_{i-1}+1)^2-1}{(b_{i-1}+1)^2+1}$$
Now notice that
$$sumlimits_{k=1}^n frac{a_k}{(1+A_k)^2}=S(1)=S(n-n+1)$$
So
$$max(sumlimits_{k=1}^n frac{a_k}{(1+A_k)^2})=max(S(n-n+1))=frac{1}{(1+b_{n-1})^2(1+A_0)}$$
$A_0=0$ since it is the sum of no variables, thus the maxima of $sumlimits_{k=1}^n frac{a_k}{(1+A_k)^2}$ can be expressed as
$$max(sumlimits_{k=1}^n frac{a_k}{(1+A_k)^2})=frac{1}{(1+b_{n-1})^2}$$
Now, let $m(n)=frac{1}{(1+b_{n-1})^2}$, it can be shown that $m(n)$ follows the recurrence relation
$$m(n+1)=frac{(m(n)+1)^2}{4}$$
So, if $c_n=1-frac{ln(n)}{2n}$ follows this property, we are done:
$$1-frac{ln(n+1)}{2(n+1)}=frac{(2-frac{ln(n)}{2n})^2}{4}$$
$$4-frac{2ln(n+1)}{n+1}=(2-frac{ln(n)}{2n})^2$$
$$4-frac{2ln(n+1)}{n+1}=4-frac{2ln(n)}{n}+frac{ln(n)^2}{4n^2}$$
$$frac{2ln(n+1)}{n+1}=frac{2ln(n)}{n}-frac{ln(n)^2}{4n^2}$$
Now this almost holds, since $frac{2ln(n+1)}{n+1} sim frac{2ln(n)}{n}$ and $frac{ln(n)^2}{4n^2}$ is a decreasing function in the interval $(e;+infty)$, and its maxima in $(1;+infty)$ is about $0.034$, in other words very very small, so $frac{2ln(n+1)}{n+1}$ comes very close to $frac{2ln(n)}{n}-frac{ln(n)^2}{4n^2}$, which explains why $1-frac{ln(n)}{2n}$ was such a good bound, additionally, it is slightly bigger than the first few values of $m(n)$ and I think that it keeps on being just a little bit larger than $m(n)$ as $n$ approaches infinity



Now to get the super sharp boundry, we should find a function satisfying $m(n+1)=frac{(m(n)+1)^2}{4}$
(On a sidenote: Happy new year!)






share|cite|improve this answer











$endgroup$









  • 1




    $begingroup$
    The upper bound $1$ can be shown by same method as here, but my main problem was establishing $c_n = left(1-frac{log n}{2n}right)$ :-)
    $endgroup$
    – r9m
    Dec 31 '14 at 11:49






  • 1




    $begingroup$
    I nderstand, I'm working on it right now :) Thanks for the link.
    $endgroup$
    – cirpis
    Dec 31 '14 at 11:52










  • $begingroup$
    Also I'm not sure if the improved bound is stronger than Cauchy-Schwarz or not .. its just a possibility. I wasn't able to find a counter example though ! ^_^
    $endgroup$
    – r9m
    Dec 31 '14 at 11:52






  • 1




    $begingroup$
    Solved the "stronger" version. Sorry if this is too messy, but I think this shows the general idea clearly enough. Haven't had any progress with the other inequality though, I will try to use a similiar argument as I did here in a bit. Cheers!
    $endgroup$
    – cirpis
    Jan 1 '15 at 0:08










  • $begingroup$
    Happy New Year to you too :-) and thanks a lot for the wonderful answer !! :D
    $endgroup$
    – r9m
    Jan 1 '15 at 5:30












Your Answer





StackExchange.ifUsing("editor", function () {
return StackExchange.using("mathjaxEditing", function () {
StackExchange.MarkdownEditor.creationCallbacks.add(function (editor, postfix) {
StackExchange.mathjaxEditing.prepareWmdForMathJax(editor, postfix, [["$", "$"], ["\\(","\\)"]]);
});
});
}, "mathjax-editing");

StackExchange.ready(function() {
var channelOptions = {
tags: "".split(" "),
id: "69"
};
initTagRenderer("".split(" "), "".split(" "), channelOptions);

StackExchange.using("externalEditor", function() {
// Have to fire editor after snippets, if snippets enabled
if (StackExchange.settings.snippets.snippetsEnabled) {
StackExchange.using("snippets", function() {
createEditor();
});
}
else {
createEditor();
}
});

function createEditor() {
StackExchange.prepareEditor({
heartbeatType: 'answer',
autoActivateHeartbeat: false,
convertImagesToLinks: true,
noModals: true,
showLowRepImageUploadWarning: true,
reputationToPostImages: 10,
bindNavPrevention: true,
postfix: "",
imageUploader: {
brandingHtml: "Powered by u003ca class="icon-imgur-white" href="https://imgur.com/"u003eu003c/au003e",
contentPolicyHtml: "User contributions licensed under u003ca href="https://creativecommons.org/licenses/by-sa/3.0/"u003ecc by-sa 3.0 with attribution requiredu003c/au003e u003ca href="https://stackoverflow.com/legal/content-policy"u003e(content policy)u003c/au003e",
allowUrls: true
},
noCode: true, onDemand: true,
discardSelector: ".discard-answer"
,immediatelyShowMarkdownHelp:true
});


}
});














draft saved

draft discarded


















StackExchange.ready(
function () {
StackExchange.openid.initPostLogin('.new-post-login', 'https%3a%2f%2fmath.stackexchange.com%2fquestions%2f853354%2fstronger-version-of-amm-problem-11145-april-2005%23new-answer', 'question_page');
}
);

Post as a guest















Required, but never shown

























1 Answer
1






active

oldest

votes








1 Answer
1






active

oldest

votes









active

oldest

votes






active

oldest

votes









10





+100







$begingroup$

Haven't had any progress with the original problem but I have done significant progress on the "sister" problem:



Let $A_k=sumlimits_{i=1}^k a_i $, thus the problem can be re-written as
$$sumlimits_{k=1}^n frac{1}{1+A_k} leq sqrt{sumlimits_{k=1}^n frac{1}{a_k}}$$
or, since both sides are positive,
$$left( sumlimits_{k=1}^n frac{1}{1+A_k} right)^2 leq {sumlimits_{k=1}^n frac{1}{a_k}}$$
Now this is very similiar to the Cauchy-Schwarz inequality, that is, for any positive integers $x_1,x_2...x_n$ and $y_1,y_2...y_n$, the following inequality holds:
$$left( sumlimits_{k=1}^n x_ky_k right)^2 leq left( sumlimits_{k=1}^n x_k^2 right)left( sumlimits_{k=1}^n y_k^2 right)$$
So, assuming $x_k=frac{1}{sqrt{a_k}}$, we gain that $y_k=frac{sqrt{a_k}}{1+A_k}$,
now if $ sumlimits_{k=1}^n y_k^2 =sumlimits_{k=1}^n frac{a_k}{(1+A_k)^2}leq1$
then the original inequality holds. A very nice proof of this due to r9m himself can be found here.



In case of the stronger version
$$sumlimits_{k=1}^n frac{1}{1+A_k} leq c_nsqrt{sumlimits_{k=1}^n frac{1}{a_k}}$$
Using the Cauchy-Schwarz inequality in a similiar fashion we gain that in order for the original inequality to hold,
$$sumlimits_{k=1}^n frac{a_k}{(1+A_k)^2}leq c_n$$ must also hold.
To prove this let us look at the maximal values of $sumlimits_{k=1}^n frac{a_k}{(1+A_k)^2}$. To obtain the maxima we must solve a system of $n$ partial derivatives of this sum equal to zero, or notationally:



begin{cases}
&frac{partial}{partial a_1}sumlimits_{k=1}^n frac{a_k}{(1+A_k)^2}=0 \
&frac{partial}{partial a_2}sumlimits_{k=1}^n frac{a_k}{(1+A_k)^2}=0\
&.\
&.\
&frac{partial}{partial a_n}sumlimits_{k=1}^n frac{a_k}{(1+A_k)^2}=0
end{cases}



Note that if we define $S(i)=sumlimits_{k=i}^n frac{a_k}{(1+A_k)^2}$, then, since no terms of our sum with index less than $i$ contain $a_i$, we may rewrite the system as:
begin{cases}
&frac{partial}{partial a_1}S(1)=0 \
&frac{partial}{partial a_2}S(2)=0\
&.\
&.\
&frac{partial}{partial a_n}S(n)=0
end{cases}



Note that $S(n)= frac{a_n}{(1+A_{n-1}+a_n)^2}$, thus $$frac{partial}{partial a_n}S(n)=frac{1+A_{n-1}-a_n}{(1+A_{n-1}+a_n)^3}$$
Note that it is zero if $a_n=1+A_{n-1}$
Now let us show that in order for maxima to be gained $a_{i}=b_{n-i}(1+A_{i-1})$ for all natural $i<n$ with some sequence $b_i$. So we just gained that $a_{n}=b_0(1+A_{n-2})$ with $b_0=1$.
The maximal value of $S(n)$ is thus
$$max(S(n))=max(frac{a_n}{(1+A_{n-1}+a_n)^2})=frac{b_0(1+A_{n-1})}{(1+A_{n-1}+b_0(1+A_{n-1}))^2}=frac{b_0}{(1+b_0)^2(1+A_{n-1}))}$$



We may now simplify $S(n-1)$, since we know that $a_n=b_0(1+A_{n-1})$,
$$S(n-1)=frac{a_{n-1}}{(1+A_{n-2}+a_{n-1})^2}+S(n)=frac{a_{n-1}}{(1+A_{n-2}+a_{n-1})^2}+frac{a_n}{(1+A_{n-1}+a_n)^2}=frac{a_{n-1}}{(1+A_{n-2}+a_{n-1})^2}+frac{b_0}{(b_0+1)^2(1+A_{n-2}+a_{n-1})}=frac{((b_0+1)^2+b_0)a_{n-1}+b_0(1+A_{n-2})}{(b_0+1)^2(1+A_{n-2}+a_{n-1})^2}$$



So the partial derivative:
$$frac{partial}{partial a_{n-1}}S(n-1)=frac{partial}{partial a_{n-1}}frac{((b_0+1)^2+b_0)a_{n-1}+b_0(1+A_{n-2})}{(b_0+1)^2(1+A_{n-2}+a_{n-1})^2}=frac{((b_0+1)^2-b_0)(1+A_{n-2})-((b_0+1)^2+b_0)a_{n-1}}{(1+b_0)^2(1+A_{n-2}+a_{n-1})^3}$$



Thus for maxima to be atained, $a_{n-1}=b_1(1+A_{n-2})$, and $b_1=frac{(b_0+1)^2-b_0}{(b_0+1)^2+b_0}$ (particularly, $b_1=frac 3 5$),



The maximal value of $S(n-1)$ is then
$$max(S(n-1))=max(frac{((b_0+1)^2+b_0)a_{n-1}+b_0(1+A_{n-2})}{(1+b_0)^2(1+A_{n-2}+a_{n-1})^2})=frac{((b_0+1)^2-b_0)(1+A_{n-2})+b_0(1+A_{n-2})}{(b_0+1)^2(1+A_{n-2}+b_1(1+A_{n-2}))^2}=frac{(b_0+1)^2(1+A_{n-2})}{(b_0+1)^2(b_1+1)^2(1+A_{n-2})^2}=frac{1}{(b_1+1)^2(1+A_{n-2})}$$



$S(n-2)$ can then be calculated as
$$S(n-2)=frac{a_{n-2}}{(1+A_{n-3}+a_{n-2})^2}+S(n-1)=frac{a_{n-2}}{(1+A_{n-3}+a_{n-2})^2}+frac{1}{(b_1+1)^2(1+A_{n-3}+a_{n-2})}=frac{1+A_{n-3}+(1+(b_1+1)^2)a_{n-2}}{(b_1+1)^2(1+A_{n-3}+a_{n-2})^2}$$



The partial derivative is thus:
$$frac{partial}{partial a_{n-2}}S(n-2)=frac{partial}{partial a_{n-2}}frac{1+A_{n-3}+(1+(b_1+1)^2)a_{n-2}}{(b_1+1)^2(1+A_{n-3}+a_{n-2})^2}=frac{b_1(b_1+1)(1+A_{n-3})-(1+(b_1+1)^2)a_{n-2}}{(b_1+1)^2(1+A_{n-3}+a_{n-2})^3}$$



So, once again to attain maxima $a_{n-2}=b_2(1+A_{n-3})$ and $b_2=frac{(b_1+1)^2-1}{(b_1+1)^2+1}$.
And the maximum of $S(n-2)$ is
$$max(S(n-2))=max(frac{1+A_{n-3}+(1+(b_1+1)^2)a_{n-2}}{(b_1+1)^2(1+A_{n-3}+a_{n-2})^2})=frac{((b_1+1)^2-1)(1+A_{n-3})+1+A_{n-3}}{(b_1+1)^2(1+A_{n-3}+b_2(1+A_{n-2}))^2}=frac{(b_1+1)^2(1+A_{n-3})}{(b_1+1)^2(b_2+1)^2(1+A_{n-3})^2}=frac{1}{(b_2+1)^2(1+A_{n-3})}$$



Notice that the expression for the maximum value of $S(n-2)$ is almost identical to the one with $S(n-1)$, only all the indices are shifted by one, thus all the following operations that will be done on higher indices will be analogous, thus we can say that in general
$$max(S(n-i))=frac{1}{(1+b_i)^2(1+A_{n-i-1})}$$
$$b_i=frac{(b_{i-1}+1)^2-1}{(b_{i-1}+1)^2+1}$$
Now notice that
$$sumlimits_{k=1}^n frac{a_k}{(1+A_k)^2}=S(1)=S(n-n+1)$$
So
$$max(sumlimits_{k=1}^n frac{a_k}{(1+A_k)^2})=max(S(n-n+1))=frac{1}{(1+b_{n-1})^2(1+A_0)}$$
$A_0=0$ since it is the sum of no variables, thus the maxima of $sumlimits_{k=1}^n frac{a_k}{(1+A_k)^2}$ can be expressed as
$$max(sumlimits_{k=1}^n frac{a_k}{(1+A_k)^2})=frac{1}{(1+b_{n-1})^2}$$
Now, let $m(n)=frac{1}{(1+b_{n-1})^2}$, it can be shown that $m(n)$ follows the recurrence relation
$$m(n+1)=frac{(m(n)+1)^2}{4}$$
So, if $c_n=1-frac{ln(n)}{2n}$ follows this property, we are done:
$$1-frac{ln(n+1)}{2(n+1)}=frac{(2-frac{ln(n)}{2n})^2}{4}$$
$$4-frac{2ln(n+1)}{n+1}=(2-frac{ln(n)}{2n})^2$$
$$4-frac{2ln(n+1)}{n+1}=4-frac{2ln(n)}{n}+frac{ln(n)^2}{4n^2}$$
$$frac{2ln(n+1)}{n+1}=frac{2ln(n)}{n}-frac{ln(n)^2}{4n^2}$$
Now this almost holds, since $frac{2ln(n+1)}{n+1} sim frac{2ln(n)}{n}$ and $frac{ln(n)^2}{4n^2}$ is a decreasing function in the interval $(e;+infty)$, and its maxima in $(1;+infty)$ is about $0.034$, in other words very very small, so $frac{2ln(n+1)}{n+1}$ comes very close to $frac{2ln(n)}{n}-frac{ln(n)^2}{4n^2}$, which explains why $1-frac{ln(n)}{2n}$ was such a good bound, additionally, it is slightly bigger than the first few values of $m(n)$ and I think that it keeps on being just a little bit larger than $m(n)$ as $n$ approaches infinity



Now to get the super sharp boundry, we should find a function satisfying $m(n+1)=frac{(m(n)+1)^2}{4}$
(On a sidenote: Happy new year!)






share|cite|improve this answer











$endgroup$









  • 1




    $begingroup$
    The upper bound $1$ can be shown by same method as here, but my main problem was establishing $c_n = left(1-frac{log n}{2n}right)$ :-)
    $endgroup$
    – r9m
    Dec 31 '14 at 11:49






  • 1




    $begingroup$
    I nderstand, I'm working on it right now :) Thanks for the link.
    $endgroup$
    – cirpis
    Dec 31 '14 at 11:52










  • $begingroup$
    Also I'm not sure if the improved bound is stronger than Cauchy-Schwarz or not .. its just a possibility. I wasn't able to find a counter example though ! ^_^
    $endgroup$
    – r9m
    Dec 31 '14 at 11:52






  • 1




    $begingroup$
    Solved the "stronger" version. Sorry if this is too messy, but I think this shows the general idea clearly enough. Haven't had any progress with the other inequality though, I will try to use a similiar argument as I did here in a bit. Cheers!
    $endgroup$
    – cirpis
    Jan 1 '15 at 0:08










  • $begingroup$
    Happy New Year to you too :-) and thanks a lot for the wonderful answer !! :D
    $endgroup$
    – r9m
    Jan 1 '15 at 5:30
















10





+100







$begingroup$

Haven't had any progress with the original problem but I have done significant progress on the "sister" problem:



Let $A_k=sumlimits_{i=1}^k a_i $, thus the problem can be re-written as
$$sumlimits_{k=1}^n frac{1}{1+A_k} leq sqrt{sumlimits_{k=1}^n frac{1}{a_k}}$$
or, since both sides are positive,
$$left( sumlimits_{k=1}^n frac{1}{1+A_k} right)^2 leq {sumlimits_{k=1}^n frac{1}{a_k}}$$
Now this is very similiar to the Cauchy-Schwarz inequality, that is, for any positive integers $x_1,x_2...x_n$ and $y_1,y_2...y_n$, the following inequality holds:
$$left( sumlimits_{k=1}^n x_ky_k right)^2 leq left( sumlimits_{k=1}^n x_k^2 right)left( sumlimits_{k=1}^n y_k^2 right)$$
So, assuming $x_k=frac{1}{sqrt{a_k}}$, we gain that $y_k=frac{sqrt{a_k}}{1+A_k}$,
now if $ sumlimits_{k=1}^n y_k^2 =sumlimits_{k=1}^n frac{a_k}{(1+A_k)^2}leq1$
then the original inequality holds. A very nice proof of this due to r9m himself can be found here.



In case of the stronger version
$$sumlimits_{k=1}^n frac{1}{1+A_k} leq c_nsqrt{sumlimits_{k=1}^n frac{1}{a_k}}$$
Using the Cauchy-Schwarz inequality in a similiar fashion we gain that in order for the original inequality to hold,
$$sumlimits_{k=1}^n frac{a_k}{(1+A_k)^2}leq c_n$$ must also hold.
To prove this let us look at the maximal values of $sumlimits_{k=1}^n frac{a_k}{(1+A_k)^2}$. To obtain the maxima we must solve a system of $n$ partial derivatives of this sum equal to zero, or notationally:



begin{cases}
&frac{partial}{partial a_1}sumlimits_{k=1}^n frac{a_k}{(1+A_k)^2}=0 \
&frac{partial}{partial a_2}sumlimits_{k=1}^n frac{a_k}{(1+A_k)^2}=0\
&.\
&.\
&frac{partial}{partial a_n}sumlimits_{k=1}^n frac{a_k}{(1+A_k)^2}=0
end{cases}



Note that if we define $S(i)=sumlimits_{k=i}^n frac{a_k}{(1+A_k)^2}$, then, since no terms of our sum with index less than $i$ contain $a_i$, we may rewrite the system as:
begin{cases}
&frac{partial}{partial a_1}S(1)=0 \
&frac{partial}{partial a_2}S(2)=0\
&.\
&.\
&frac{partial}{partial a_n}S(n)=0
end{cases}



Note that $S(n)= frac{a_n}{(1+A_{n-1}+a_n)^2}$, thus $$frac{partial}{partial a_n}S(n)=frac{1+A_{n-1}-a_n}{(1+A_{n-1}+a_n)^3}$$
Note that it is zero if $a_n=1+A_{n-1}$
Now let us show that in order for maxima to be gained $a_{i}=b_{n-i}(1+A_{i-1})$ for all natural $i<n$ with some sequence $b_i$. So we just gained that $a_{n}=b_0(1+A_{n-2})$ with $b_0=1$.
The maximal value of $S(n)$ is thus
$$max(S(n))=max(frac{a_n}{(1+A_{n-1}+a_n)^2})=frac{b_0(1+A_{n-1})}{(1+A_{n-1}+b_0(1+A_{n-1}))^2}=frac{b_0}{(1+b_0)^2(1+A_{n-1}))}$$



We may now simplify $S(n-1)$, since we know that $a_n=b_0(1+A_{n-1})$,
$$S(n-1)=frac{a_{n-1}}{(1+A_{n-2}+a_{n-1})^2}+S(n)=frac{a_{n-1}}{(1+A_{n-2}+a_{n-1})^2}+frac{a_n}{(1+A_{n-1}+a_n)^2}=frac{a_{n-1}}{(1+A_{n-2}+a_{n-1})^2}+frac{b_0}{(b_0+1)^2(1+A_{n-2}+a_{n-1})}=frac{((b_0+1)^2+b_0)a_{n-1}+b_0(1+A_{n-2})}{(b_0+1)^2(1+A_{n-2}+a_{n-1})^2}$$



So the partial derivative:
$$frac{partial}{partial a_{n-1}}S(n-1)=frac{partial}{partial a_{n-1}}frac{((b_0+1)^2+b_0)a_{n-1}+b_0(1+A_{n-2})}{(b_0+1)^2(1+A_{n-2}+a_{n-1})^2}=frac{((b_0+1)^2-b_0)(1+A_{n-2})-((b_0+1)^2+b_0)a_{n-1}}{(1+b_0)^2(1+A_{n-2}+a_{n-1})^3}$$



Thus for maxima to be atained, $a_{n-1}=b_1(1+A_{n-2})$, and $b_1=frac{(b_0+1)^2-b_0}{(b_0+1)^2+b_0}$ (particularly, $b_1=frac 3 5$),



The maximal value of $S(n-1)$ is then
$$max(S(n-1))=max(frac{((b_0+1)^2+b_0)a_{n-1}+b_0(1+A_{n-2})}{(1+b_0)^2(1+A_{n-2}+a_{n-1})^2})=frac{((b_0+1)^2-b_0)(1+A_{n-2})+b_0(1+A_{n-2})}{(b_0+1)^2(1+A_{n-2}+b_1(1+A_{n-2}))^2}=frac{(b_0+1)^2(1+A_{n-2})}{(b_0+1)^2(b_1+1)^2(1+A_{n-2})^2}=frac{1}{(b_1+1)^2(1+A_{n-2})}$$



$S(n-2)$ can then be calculated as
$$S(n-2)=frac{a_{n-2}}{(1+A_{n-3}+a_{n-2})^2}+S(n-1)=frac{a_{n-2}}{(1+A_{n-3}+a_{n-2})^2}+frac{1}{(b_1+1)^2(1+A_{n-3}+a_{n-2})}=frac{1+A_{n-3}+(1+(b_1+1)^2)a_{n-2}}{(b_1+1)^2(1+A_{n-3}+a_{n-2})^2}$$



The partial derivative is thus:
$$frac{partial}{partial a_{n-2}}S(n-2)=frac{partial}{partial a_{n-2}}frac{1+A_{n-3}+(1+(b_1+1)^2)a_{n-2}}{(b_1+1)^2(1+A_{n-3}+a_{n-2})^2}=frac{b_1(b_1+1)(1+A_{n-3})-(1+(b_1+1)^2)a_{n-2}}{(b_1+1)^2(1+A_{n-3}+a_{n-2})^3}$$



So, once again to attain maxima $a_{n-2}=b_2(1+A_{n-3})$ and $b_2=frac{(b_1+1)^2-1}{(b_1+1)^2+1}$.
And the maximum of $S(n-2)$ is
$$max(S(n-2))=max(frac{1+A_{n-3}+(1+(b_1+1)^2)a_{n-2}}{(b_1+1)^2(1+A_{n-3}+a_{n-2})^2})=frac{((b_1+1)^2-1)(1+A_{n-3})+1+A_{n-3}}{(b_1+1)^2(1+A_{n-3}+b_2(1+A_{n-2}))^2}=frac{(b_1+1)^2(1+A_{n-3})}{(b_1+1)^2(b_2+1)^2(1+A_{n-3})^2}=frac{1}{(b_2+1)^2(1+A_{n-3})}$$



Notice that the expression for the maximum value of $S(n-2)$ is almost identical to the one with $S(n-1)$, only all the indices are shifted by one, thus all the following operations that will be done on higher indices will be analogous, thus we can say that in general
$$max(S(n-i))=frac{1}{(1+b_i)^2(1+A_{n-i-1})}$$
$$b_i=frac{(b_{i-1}+1)^2-1}{(b_{i-1}+1)^2+1}$$
Now notice that
$$sumlimits_{k=1}^n frac{a_k}{(1+A_k)^2}=S(1)=S(n-n+1)$$
So
$$max(sumlimits_{k=1}^n frac{a_k}{(1+A_k)^2})=max(S(n-n+1))=frac{1}{(1+b_{n-1})^2(1+A_0)}$$
$A_0=0$ since it is the sum of no variables, thus the maxima of $sumlimits_{k=1}^n frac{a_k}{(1+A_k)^2}$ can be expressed as
$$max(sumlimits_{k=1}^n frac{a_k}{(1+A_k)^2})=frac{1}{(1+b_{n-1})^2}$$
Now, let $m(n)=frac{1}{(1+b_{n-1})^2}$, it can be shown that $m(n)$ follows the recurrence relation
$$m(n+1)=frac{(m(n)+1)^2}{4}$$
So, if $c_n=1-frac{ln(n)}{2n}$ follows this property, we are done:
$$1-frac{ln(n+1)}{2(n+1)}=frac{(2-frac{ln(n)}{2n})^2}{4}$$
$$4-frac{2ln(n+1)}{n+1}=(2-frac{ln(n)}{2n})^2$$
$$4-frac{2ln(n+1)}{n+1}=4-frac{2ln(n)}{n}+frac{ln(n)^2}{4n^2}$$
$$frac{2ln(n+1)}{n+1}=frac{2ln(n)}{n}-frac{ln(n)^2}{4n^2}$$
Now this almost holds, since $frac{2ln(n+1)}{n+1} sim frac{2ln(n)}{n}$ and $frac{ln(n)^2}{4n^2}$ is a decreasing function in the interval $(e;+infty)$, and its maxima in $(1;+infty)$ is about $0.034$, in other words very very small, so $frac{2ln(n+1)}{n+1}$ comes very close to $frac{2ln(n)}{n}-frac{ln(n)^2}{4n^2}$, which explains why $1-frac{ln(n)}{2n}$ was such a good bound, additionally, it is slightly bigger than the first few values of $m(n)$ and I think that it keeps on being just a little bit larger than $m(n)$ as $n$ approaches infinity



Now to get the super sharp boundry, we should find a function satisfying $m(n+1)=frac{(m(n)+1)^2}{4}$
(On a sidenote: Happy new year!)






share|cite|improve this answer











$endgroup$









  • 1




    $begingroup$
    The upper bound $1$ can be shown by same method as here, but my main problem was establishing $c_n = left(1-frac{log n}{2n}right)$ :-)
    $endgroup$
    – r9m
    Dec 31 '14 at 11:49






  • 1




    $begingroup$
    I nderstand, I'm working on it right now :) Thanks for the link.
    $endgroup$
    – cirpis
    Dec 31 '14 at 11:52










  • $begingroup$
    Also I'm not sure if the improved bound is stronger than Cauchy-Schwarz or not .. its just a possibility. I wasn't able to find a counter example though ! ^_^
    $endgroup$
    – r9m
    Dec 31 '14 at 11:52






  • 1




    $begingroup$
    Solved the "stronger" version. Sorry if this is too messy, but I think this shows the general idea clearly enough. Haven't had any progress with the other inequality though, I will try to use a similiar argument as I did here in a bit. Cheers!
    $endgroup$
    – cirpis
    Jan 1 '15 at 0:08










  • $begingroup$
    Happy New Year to you too :-) and thanks a lot for the wonderful answer !! :D
    $endgroup$
    – r9m
    Jan 1 '15 at 5:30














10





+100







10





+100



10




+100



$begingroup$

Haven't had any progress with the original problem but I have done significant progress on the "sister" problem:



Let $A_k=sumlimits_{i=1}^k a_i $, thus the problem can be re-written as
$$sumlimits_{k=1}^n frac{1}{1+A_k} leq sqrt{sumlimits_{k=1}^n frac{1}{a_k}}$$
or, since both sides are positive,
$$left( sumlimits_{k=1}^n frac{1}{1+A_k} right)^2 leq {sumlimits_{k=1}^n frac{1}{a_k}}$$
Now this is very similiar to the Cauchy-Schwarz inequality, that is, for any positive integers $x_1,x_2...x_n$ and $y_1,y_2...y_n$, the following inequality holds:
$$left( sumlimits_{k=1}^n x_ky_k right)^2 leq left( sumlimits_{k=1}^n x_k^2 right)left( sumlimits_{k=1}^n y_k^2 right)$$
So, assuming $x_k=frac{1}{sqrt{a_k}}$, we gain that $y_k=frac{sqrt{a_k}}{1+A_k}$,
now if $ sumlimits_{k=1}^n y_k^2 =sumlimits_{k=1}^n frac{a_k}{(1+A_k)^2}leq1$
then the original inequality holds. A very nice proof of this due to r9m himself can be found here.



In case of the stronger version
$$sumlimits_{k=1}^n frac{1}{1+A_k} leq c_nsqrt{sumlimits_{k=1}^n frac{1}{a_k}}$$
Using the Cauchy-Schwarz inequality in a similiar fashion we gain that in order for the original inequality to hold,
$$sumlimits_{k=1}^n frac{a_k}{(1+A_k)^2}leq c_n$$ must also hold.
To prove this let us look at the maximal values of $sumlimits_{k=1}^n frac{a_k}{(1+A_k)^2}$. To obtain the maxima we must solve a system of $n$ partial derivatives of this sum equal to zero, or notationally:



begin{cases}
&frac{partial}{partial a_1}sumlimits_{k=1}^n frac{a_k}{(1+A_k)^2}=0 \
&frac{partial}{partial a_2}sumlimits_{k=1}^n frac{a_k}{(1+A_k)^2}=0\
&.\
&.\
&frac{partial}{partial a_n}sumlimits_{k=1}^n frac{a_k}{(1+A_k)^2}=0
end{cases}



Note that if we define $S(i)=sumlimits_{k=i}^n frac{a_k}{(1+A_k)^2}$, then, since no terms of our sum with index less than $i$ contain $a_i$, we may rewrite the system as:
begin{cases}
&frac{partial}{partial a_1}S(1)=0 \
&frac{partial}{partial a_2}S(2)=0\
&.\
&.\
&frac{partial}{partial a_n}S(n)=0
end{cases}



Note that $S(n)= frac{a_n}{(1+A_{n-1}+a_n)^2}$, thus $$frac{partial}{partial a_n}S(n)=frac{1+A_{n-1}-a_n}{(1+A_{n-1}+a_n)^3}$$
Note that it is zero if $a_n=1+A_{n-1}$
Now let us show that in order for maxima to be gained $a_{i}=b_{n-i}(1+A_{i-1})$ for all natural $i<n$ with some sequence $b_i$. So we just gained that $a_{n}=b_0(1+A_{n-2})$ with $b_0=1$.
The maximal value of $S(n)$ is thus
$$max(S(n))=max(frac{a_n}{(1+A_{n-1}+a_n)^2})=frac{b_0(1+A_{n-1})}{(1+A_{n-1}+b_0(1+A_{n-1}))^2}=frac{b_0}{(1+b_0)^2(1+A_{n-1}))}$$



We may now simplify $S(n-1)$, since we know that $a_n=b_0(1+A_{n-1})$,
$$S(n-1)=frac{a_{n-1}}{(1+A_{n-2}+a_{n-1})^2}+S(n)=frac{a_{n-1}}{(1+A_{n-2}+a_{n-1})^2}+frac{a_n}{(1+A_{n-1}+a_n)^2}=frac{a_{n-1}}{(1+A_{n-2}+a_{n-1})^2}+frac{b_0}{(b_0+1)^2(1+A_{n-2}+a_{n-1})}=frac{((b_0+1)^2+b_0)a_{n-1}+b_0(1+A_{n-2})}{(b_0+1)^2(1+A_{n-2}+a_{n-1})^2}$$



So the partial derivative:
$$frac{partial}{partial a_{n-1}}S(n-1)=frac{partial}{partial a_{n-1}}frac{((b_0+1)^2+b_0)a_{n-1}+b_0(1+A_{n-2})}{(b_0+1)^2(1+A_{n-2}+a_{n-1})^2}=frac{((b_0+1)^2-b_0)(1+A_{n-2})-((b_0+1)^2+b_0)a_{n-1}}{(1+b_0)^2(1+A_{n-2}+a_{n-1})^3}$$



Thus for maxima to be atained, $a_{n-1}=b_1(1+A_{n-2})$, and $b_1=frac{(b_0+1)^2-b_0}{(b_0+1)^2+b_0}$ (particularly, $b_1=frac 3 5$),



The maximal value of $S(n-1)$ is then
$$max(S(n-1))=max(frac{((b_0+1)^2+b_0)a_{n-1}+b_0(1+A_{n-2})}{(1+b_0)^2(1+A_{n-2}+a_{n-1})^2})=frac{((b_0+1)^2-b_0)(1+A_{n-2})+b_0(1+A_{n-2})}{(b_0+1)^2(1+A_{n-2}+b_1(1+A_{n-2}))^2}=frac{(b_0+1)^2(1+A_{n-2})}{(b_0+1)^2(b_1+1)^2(1+A_{n-2})^2}=frac{1}{(b_1+1)^2(1+A_{n-2})}$$



$S(n-2)$ can then be calculated as
$$S(n-2)=frac{a_{n-2}}{(1+A_{n-3}+a_{n-2})^2}+S(n-1)=frac{a_{n-2}}{(1+A_{n-3}+a_{n-2})^2}+frac{1}{(b_1+1)^2(1+A_{n-3}+a_{n-2})}=frac{1+A_{n-3}+(1+(b_1+1)^2)a_{n-2}}{(b_1+1)^2(1+A_{n-3}+a_{n-2})^2}$$



The partial derivative is thus:
$$frac{partial}{partial a_{n-2}}S(n-2)=frac{partial}{partial a_{n-2}}frac{1+A_{n-3}+(1+(b_1+1)^2)a_{n-2}}{(b_1+1)^2(1+A_{n-3}+a_{n-2})^2}=frac{b_1(b_1+1)(1+A_{n-3})-(1+(b_1+1)^2)a_{n-2}}{(b_1+1)^2(1+A_{n-3}+a_{n-2})^3}$$



So, once again to attain maxima $a_{n-2}=b_2(1+A_{n-3})$ and $b_2=frac{(b_1+1)^2-1}{(b_1+1)^2+1}$.
And the maximum of $S(n-2)$ is
$$max(S(n-2))=max(frac{1+A_{n-3}+(1+(b_1+1)^2)a_{n-2}}{(b_1+1)^2(1+A_{n-3}+a_{n-2})^2})=frac{((b_1+1)^2-1)(1+A_{n-3})+1+A_{n-3}}{(b_1+1)^2(1+A_{n-3}+b_2(1+A_{n-2}))^2}=frac{(b_1+1)^2(1+A_{n-3})}{(b_1+1)^2(b_2+1)^2(1+A_{n-3})^2}=frac{1}{(b_2+1)^2(1+A_{n-3})}$$



Notice that the expression for the maximum value of $S(n-2)$ is almost identical to the one with $S(n-1)$, only all the indices are shifted by one, thus all the following operations that will be done on higher indices will be analogous, thus we can say that in general
$$max(S(n-i))=frac{1}{(1+b_i)^2(1+A_{n-i-1})}$$
$$b_i=frac{(b_{i-1}+1)^2-1}{(b_{i-1}+1)^2+1}$$
Now notice that
$$sumlimits_{k=1}^n frac{a_k}{(1+A_k)^2}=S(1)=S(n-n+1)$$
So
$$max(sumlimits_{k=1}^n frac{a_k}{(1+A_k)^2})=max(S(n-n+1))=frac{1}{(1+b_{n-1})^2(1+A_0)}$$
$A_0=0$ since it is the sum of no variables, thus the maxima of $sumlimits_{k=1}^n frac{a_k}{(1+A_k)^2}$ can be expressed as
$$max(sumlimits_{k=1}^n frac{a_k}{(1+A_k)^2})=frac{1}{(1+b_{n-1})^2}$$
Now, let $m(n)=frac{1}{(1+b_{n-1})^2}$, it can be shown that $m(n)$ follows the recurrence relation
$$m(n+1)=frac{(m(n)+1)^2}{4}$$
So, if $c_n=1-frac{ln(n)}{2n}$ follows this property, we are done:
$$1-frac{ln(n+1)}{2(n+1)}=frac{(2-frac{ln(n)}{2n})^2}{4}$$
$$4-frac{2ln(n+1)}{n+1}=(2-frac{ln(n)}{2n})^2$$
$$4-frac{2ln(n+1)}{n+1}=4-frac{2ln(n)}{n}+frac{ln(n)^2}{4n^2}$$
$$frac{2ln(n+1)}{n+1}=frac{2ln(n)}{n}-frac{ln(n)^2}{4n^2}$$
Now this almost holds, since $frac{2ln(n+1)}{n+1} sim frac{2ln(n)}{n}$ and $frac{ln(n)^2}{4n^2}$ is a decreasing function in the interval $(e;+infty)$, and its maxima in $(1;+infty)$ is about $0.034$, in other words very very small, so $frac{2ln(n+1)}{n+1}$ comes very close to $frac{2ln(n)}{n}-frac{ln(n)^2}{4n^2}$, which explains why $1-frac{ln(n)}{2n}$ was such a good bound, additionally, it is slightly bigger than the first few values of $m(n)$ and I think that it keeps on being just a little bit larger than $m(n)$ as $n$ approaches infinity



Now to get the super sharp boundry, we should find a function satisfying $m(n+1)=frac{(m(n)+1)^2}{4}$
(On a sidenote: Happy new year!)






share|cite|improve this answer











$endgroup$



Haven't had any progress with the original problem but I have done significant progress on the "sister" problem:



Let $A_k=sumlimits_{i=1}^k a_i $, thus the problem can be re-written as
$$sumlimits_{k=1}^n frac{1}{1+A_k} leq sqrt{sumlimits_{k=1}^n frac{1}{a_k}}$$
or, since both sides are positive,
$$left( sumlimits_{k=1}^n frac{1}{1+A_k} right)^2 leq {sumlimits_{k=1}^n frac{1}{a_k}}$$
Now this is very similiar to the Cauchy-Schwarz inequality, that is, for any positive integers $x_1,x_2...x_n$ and $y_1,y_2...y_n$, the following inequality holds:
$$left( sumlimits_{k=1}^n x_ky_k right)^2 leq left( sumlimits_{k=1}^n x_k^2 right)left( sumlimits_{k=1}^n y_k^2 right)$$
So, assuming $x_k=frac{1}{sqrt{a_k}}$, we gain that $y_k=frac{sqrt{a_k}}{1+A_k}$,
now if $ sumlimits_{k=1}^n y_k^2 =sumlimits_{k=1}^n frac{a_k}{(1+A_k)^2}leq1$
then the original inequality holds. A very nice proof of this due to r9m himself can be found here.



In case of the stronger version
$$sumlimits_{k=1}^n frac{1}{1+A_k} leq c_nsqrt{sumlimits_{k=1}^n frac{1}{a_k}}$$
Using the Cauchy-Schwarz inequality in a similiar fashion we gain that in order for the original inequality to hold,
$$sumlimits_{k=1}^n frac{a_k}{(1+A_k)^2}leq c_n$$ must also hold.
To prove this let us look at the maximal values of $sumlimits_{k=1}^n frac{a_k}{(1+A_k)^2}$. To obtain the maxima we must solve a system of $n$ partial derivatives of this sum equal to zero, or notationally:



begin{cases}
&frac{partial}{partial a_1}sumlimits_{k=1}^n frac{a_k}{(1+A_k)^2}=0 \
&frac{partial}{partial a_2}sumlimits_{k=1}^n frac{a_k}{(1+A_k)^2}=0\
&.\
&.\
&frac{partial}{partial a_n}sumlimits_{k=1}^n frac{a_k}{(1+A_k)^2}=0
end{cases}



Note that if we define $S(i)=sumlimits_{k=i}^n frac{a_k}{(1+A_k)^2}$, then, since no terms of our sum with index less than $i$ contain $a_i$, we may rewrite the system as:
begin{cases}
&frac{partial}{partial a_1}S(1)=0 \
&frac{partial}{partial a_2}S(2)=0\
&.\
&.\
&frac{partial}{partial a_n}S(n)=0
end{cases}



Note that $S(n)= frac{a_n}{(1+A_{n-1}+a_n)^2}$, thus $$frac{partial}{partial a_n}S(n)=frac{1+A_{n-1}-a_n}{(1+A_{n-1}+a_n)^3}$$
Note that it is zero if $a_n=1+A_{n-1}$
Now let us show that in order for maxima to be gained $a_{i}=b_{n-i}(1+A_{i-1})$ for all natural $i<n$ with some sequence $b_i$. So we just gained that $a_{n}=b_0(1+A_{n-2})$ with $b_0=1$.
The maximal value of $S(n)$ is thus
$$max(S(n))=max(frac{a_n}{(1+A_{n-1}+a_n)^2})=frac{b_0(1+A_{n-1})}{(1+A_{n-1}+b_0(1+A_{n-1}))^2}=frac{b_0}{(1+b_0)^2(1+A_{n-1}))}$$



We may now simplify $S(n-1)$, since we know that $a_n=b_0(1+A_{n-1})$,
$$S(n-1)=frac{a_{n-1}}{(1+A_{n-2}+a_{n-1})^2}+S(n)=frac{a_{n-1}}{(1+A_{n-2}+a_{n-1})^2}+frac{a_n}{(1+A_{n-1}+a_n)^2}=frac{a_{n-1}}{(1+A_{n-2}+a_{n-1})^2}+frac{b_0}{(b_0+1)^2(1+A_{n-2}+a_{n-1})}=frac{((b_0+1)^2+b_0)a_{n-1}+b_0(1+A_{n-2})}{(b_0+1)^2(1+A_{n-2}+a_{n-1})^2}$$



So the partial derivative:
$$frac{partial}{partial a_{n-1}}S(n-1)=frac{partial}{partial a_{n-1}}frac{((b_0+1)^2+b_0)a_{n-1}+b_0(1+A_{n-2})}{(b_0+1)^2(1+A_{n-2}+a_{n-1})^2}=frac{((b_0+1)^2-b_0)(1+A_{n-2})-((b_0+1)^2+b_0)a_{n-1}}{(1+b_0)^2(1+A_{n-2}+a_{n-1})^3}$$



Thus for maxima to be atained, $a_{n-1}=b_1(1+A_{n-2})$, and $b_1=frac{(b_0+1)^2-b_0}{(b_0+1)^2+b_0}$ (particularly, $b_1=frac 3 5$),



The maximal value of $S(n-1)$ is then
$$max(S(n-1))=max(frac{((b_0+1)^2+b_0)a_{n-1}+b_0(1+A_{n-2})}{(1+b_0)^2(1+A_{n-2}+a_{n-1})^2})=frac{((b_0+1)^2-b_0)(1+A_{n-2})+b_0(1+A_{n-2})}{(b_0+1)^2(1+A_{n-2}+b_1(1+A_{n-2}))^2}=frac{(b_0+1)^2(1+A_{n-2})}{(b_0+1)^2(b_1+1)^2(1+A_{n-2})^2}=frac{1}{(b_1+1)^2(1+A_{n-2})}$$



$S(n-2)$ can then be calculated as
$$S(n-2)=frac{a_{n-2}}{(1+A_{n-3}+a_{n-2})^2}+S(n-1)=frac{a_{n-2}}{(1+A_{n-3}+a_{n-2})^2}+frac{1}{(b_1+1)^2(1+A_{n-3}+a_{n-2})}=frac{1+A_{n-3}+(1+(b_1+1)^2)a_{n-2}}{(b_1+1)^2(1+A_{n-3}+a_{n-2})^2}$$



The partial derivative is thus:
$$frac{partial}{partial a_{n-2}}S(n-2)=frac{partial}{partial a_{n-2}}frac{1+A_{n-3}+(1+(b_1+1)^2)a_{n-2}}{(b_1+1)^2(1+A_{n-3}+a_{n-2})^2}=frac{b_1(b_1+1)(1+A_{n-3})-(1+(b_1+1)^2)a_{n-2}}{(b_1+1)^2(1+A_{n-3}+a_{n-2})^3}$$



So, once again to attain maxima $a_{n-2}=b_2(1+A_{n-3})$ and $b_2=frac{(b_1+1)^2-1}{(b_1+1)^2+1}$.
And the maximum of $S(n-2)$ is
$$max(S(n-2))=max(frac{1+A_{n-3}+(1+(b_1+1)^2)a_{n-2}}{(b_1+1)^2(1+A_{n-3}+a_{n-2})^2})=frac{((b_1+1)^2-1)(1+A_{n-3})+1+A_{n-3}}{(b_1+1)^2(1+A_{n-3}+b_2(1+A_{n-2}))^2}=frac{(b_1+1)^2(1+A_{n-3})}{(b_1+1)^2(b_2+1)^2(1+A_{n-3})^2}=frac{1}{(b_2+1)^2(1+A_{n-3})}$$



Notice that the expression for the maximum value of $S(n-2)$ is almost identical to the one with $S(n-1)$, only all the indices are shifted by one, thus all the following operations that will be done on higher indices will be analogous, thus we can say that in general
$$max(S(n-i))=frac{1}{(1+b_i)^2(1+A_{n-i-1})}$$
$$b_i=frac{(b_{i-1}+1)^2-1}{(b_{i-1}+1)^2+1}$$
Now notice that
$$sumlimits_{k=1}^n frac{a_k}{(1+A_k)^2}=S(1)=S(n-n+1)$$
So
$$max(sumlimits_{k=1}^n frac{a_k}{(1+A_k)^2})=max(S(n-n+1))=frac{1}{(1+b_{n-1})^2(1+A_0)}$$
$A_0=0$ since it is the sum of no variables, thus the maxima of $sumlimits_{k=1}^n frac{a_k}{(1+A_k)^2}$ can be expressed as
$$max(sumlimits_{k=1}^n frac{a_k}{(1+A_k)^2})=frac{1}{(1+b_{n-1})^2}$$
Now, let $m(n)=frac{1}{(1+b_{n-1})^2}$, it can be shown that $m(n)$ follows the recurrence relation
$$m(n+1)=frac{(m(n)+1)^2}{4}$$
So, if $c_n=1-frac{ln(n)}{2n}$ follows this property, we are done:
$$1-frac{ln(n+1)}{2(n+1)}=frac{(2-frac{ln(n)}{2n})^2}{4}$$
$$4-frac{2ln(n+1)}{n+1}=(2-frac{ln(n)}{2n})^2$$
$$4-frac{2ln(n+1)}{n+1}=4-frac{2ln(n)}{n}+frac{ln(n)^2}{4n^2}$$
$$frac{2ln(n+1)}{n+1}=frac{2ln(n)}{n}-frac{ln(n)^2}{4n^2}$$
Now this almost holds, since $frac{2ln(n+1)}{n+1} sim frac{2ln(n)}{n}$ and $frac{ln(n)^2}{4n^2}$ is a decreasing function in the interval $(e;+infty)$, and its maxima in $(1;+infty)$ is about $0.034$, in other words very very small, so $frac{2ln(n+1)}{n+1}$ comes very close to $frac{2ln(n)}{n}-frac{ln(n)^2}{4n^2}$, which explains why $1-frac{ln(n)}{2n}$ was such a good bound, additionally, it is slightly bigger than the first few values of $m(n)$ and I think that it keeps on being just a little bit larger than $m(n)$ as $n$ approaches infinity



Now to get the super sharp boundry, we should find a function satisfying $m(n+1)=frac{(m(n)+1)^2}{4}$
(On a sidenote: Happy new year!)







share|cite|improve this answer














share|cite|improve this answer



share|cite|improve this answer








edited Apr 13 '17 at 12:20









Community

1




1










answered Dec 30 '14 at 19:17









cirpiscirpis

1,656819




1,656819








  • 1




    $begingroup$
    The upper bound $1$ can be shown by same method as here, but my main problem was establishing $c_n = left(1-frac{log n}{2n}right)$ :-)
    $endgroup$
    – r9m
    Dec 31 '14 at 11:49






  • 1




    $begingroup$
    I nderstand, I'm working on it right now :) Thanks for the link.
    $endgroup$
    – cirpis
    Dec 31 '14 at 11:52










  • $begingroup$
    Also I'm not sure if the improved bound is stronger than Cauchy-Schwarz or not .. its just a possibility. I wasn't able to find a counter example though ! ^_^
    $endgroup$
    – r9m
    Dec 31 '14 at 11:52






  • 1




    $begingroup$
    Solved the "stronger" version. Sorry if this is too messy, but I think this shows the general idea clearly enough. Haven't had any progress with the other inequality though, I will try to use a similiar argument as I did here in a bit. Cheers!
    $endgroup$
    – cirpis
    Jan 1 '15 at 0:08










  • $begingroup$
    Happy New Year to you too :-) and thanks a lot for the wonderful answer !! :D
    $endgroup$
    – r9m
    Jan 1 '15 at 5:30














  • 1




    $begingroup$
    The upper bound $1$ can be shown by same method as here, but my main problem was establishing $c_n = left(1-frac{log n}{2n}right)$ :-)
    $endgroup$
    – r9m
    Dec 31 '14 at 11:49






  • 1




    $begingroup$
    I nderstand, I'm working on it right now :) Thanks for the link.
    $endgroup$
    – cirpis
    Dec 31 '14 at 11:52










  • $begingroup$
    Also I'm not sure if the improved bound is stronger than Cauchy-Schwarz or not .. its just a possibility. I wasn't able to find a counter example though ! ^_^
    $endgroup$
    – r9m
    Dec 31 '14 at 11:52






  • 1




    $begingroup$
    Solved the "stronger" version. Sorry if this is too messy, but I think this shows the general idea clearly enough. Haven't had any progress with the other inequality though, I will try to use a similiar argument as I did here in a bit. Cheers!
    $endgroup$
    – cirpis
    Jan 1 '15 at 0:08










  • $begingroup$
    Happy New Year to you too :-) and thanks a lot for the wonderful answer !! :D
    $endgroup$
    – r9m
    Jan 1 '15 at 5:30








1




1




$begingroup$
The upper bound $1$ can be shown by same method as here, but my main problem was establishing $c_n = left(1-frac{log n}{2n}right)$ :-)
$endgroup$
– r9m
Dec 31 '14 at 11:49




$begingroup$
The upper bound $1$ can be shown by same method as here, but my main problem was establishing $c_n = left(1-frac{log n}{2n}right)$ :-)
$endgroup$
– r9m
Dec 31 '14 at 11:49




1




1




$begingroup$
I nderstand, I'm working on it right now :) Thanks for the link.
$endgroup$
– cirpis
Dec 31 '14 at 11:52




$begingroup$
I nderstand, I'm working on it right now :) Thanks for the link.
$endgroup$
– cirpis
Dec 31 '14 at 11:52












$begingroup$
Also I'm not sure if the improved bound is stronger than Cauchy-Schwarz or not .. its just a possibility. I wasn't able to find a counter example though ! ^_^
$endgroup$
– r9m
Dec 31 '14 at 11:52




$begingroup$
Also I'm not sure if the improved bound is stronger than Cauchy-Schwarz or not .. its just a possibility. I wasn't able to find a counter example though ! ^_^
$endgroup$
– r9m
Dec 31 '14 at 11:52




1




1




$begingroup$
Solved the "stronger" version. Sorry if this is too messy, but I think this shows the general idea clearly enough. Haven't had any progress with the other inequality though, I will try to use a similiar argument as I did here in a bit. Cheers!
$endgroup$
– cirpis
Jan 1 '15 at 0:08




$begingroup$
Solved the "stronger" version. Sorry if this is too messy, but I think this shows the general idea clearly enough. Haven't had any progress with the other inequality though, I will try to use a similiar argument as I did here in a bit. Cheers!
$endgroup$
– cirpis
Jan 1 '15 at 0:08












$begingroup$
Happy New Year to you too :-) and thanks a lot for the wonderful answer !! :D
$endgroup$
– r9m
Jan 1 '15 at 5:30




$begingroup$
Happy New Year to you too :-) and thanks a lot for the wonderful answer !! :D
$endgroup$
– r9m
Jan 1 '15 at 5:30


















draft saved

draft discarded




















































Thanks for contributing an answer to Mathematics Stack Exchange!


  • Please be sure to answer the question. Provide details and share your research!

But avoid



  • Asking for help, clarification, or responding to other answers.

  • Making statements based on opinion; back them up with references or personal experience.


Use MathJax to format equations. MathJax reference.


To learn more, see our tips on writing great answers.




draft saved


draft discarded














StackExchange.ready(
function () {
StackExchange.openid.initPostLogin('.new-post-login', 'https%3a%2f%2fmath.stackexchange.com%2fquestions%2f853354%2fstronger-version-of-amm-problem-11145-april-2005%23new-answer', 'question_page');
}
);

Post as a guest















Required, but never shown





















































Required, but never shown














Required, but never shown












Required, but never shown







Required, but never shown

































Required, but never shown














Required, but never shown












Required, but never shown







Required, but never shown







Popular posts from this blog

Le Mesnil-Réaume

Ida-Boy-Ed-Garten

web3.py web3.isConnected() returns false always